WBR0990: Difference between revisions

Jump to navigation Jump to search
No edit summary
m (refreshing WBR questions)
 
(3 intermediate revisions by 2 users not shown)
Line 1: Line 1:
{{WBRQuestion
{{WBRQuestion
|QuestionAuthor=William J Gibson
|QuestionAuthor=William J Gibson (Reviewed by Serge Korjian)
|ExamType=USMLE Step 1
|ExamType=USMLE Step 1
|MainCategory=Physiology
|MainCategory=Physiology
Line 21: Line 21:
|MainCategory=Physiology
|MainCategory=Physiology
|SubCategory=Pulmonology
|SubCategory=Pulmonology
|Prompt=An attending in the cardiac intensive care unit at major academic medical center is teaching a resident about the pulmonary vasculature.  A patient with a history of hypertrophic cardiomyopathy is recovering from an anterior myocardial infarction and the following values are measured.
|Prompt=An attending in the cardiac intensive care unit at a major academic medical center is teaching a resident about the pulmonary vasculature.  A patient with a history of hypertrophic cardiomyopathy is recovering from an anterior myocardial infarction and the following values below are measured. Which of the following corresponds to the pulmonary vascular resistance in this patient?
<br>
'''Pulmonary artery pressure (mean): 15 mmHg'''<br>
'''Pulmonary capillary wedge pressure (mean):  10 mmHg'''<br>
'''Aortic systolic pressure: 140 mmHg'''<br>
'''Aortic diastolic pressure: 90 mmHg'''<br>
'''Heart rate: 90/min'''<br>
'''Stroke volume: 40 mL'''<br>
|Explanation=The pulmonary vascular resistance of a vessel can be calculated by the change in pressure across the segment divided by the flow (R=P/Q).  Generally, the pulmonary capillary wedge pressure is a good estimate of the pressure in the left atrium.  Therefore, pulmonary vascular resistance is given by the formula:


Pulmonary artery pressure (mean): 15
[[File:PVREqn101814.png]]
Pulmonary capillary wedge pressure (mean):  10
Aortic systolic pressure: 140
Aortic diastolic pressure: 90
Heart rate: 90 per minute
Stroke Volume: 40 mL
 
Which of the following corresponds to the pulmonary vascular resistance in this patient?
|Explanation=In this question we are asked to calculate the pulmonary vascular resistance for this patient.  Recall that the resistance of a vessel can be calculated by the change in pressure across the segment divided by the flow (R=P/Q).  Generally, the pulmonary capillary wedge pressure is a good estimate of the pressure in the left atrium. Therefore, pulmonary vascular resistance is given by the formula:
 
<math>PVR = (P_{pulmonary artery} - P_{capillary wedge})/Cardiac Output</math>
 
{{math|<VAR>&alpha;</VAR>}}


Plugging in values from the above case we get the following:
Plugging in values from the above case we get the following:
(15 mmHg - 10 mmHg)/(90 beats/minute*(.040 L/min))=5/3.60=1.39 mmHg·min/L.
(15 mmHg - 10 mmHg)/(90 beats/minute*(.040 L/min))=5/3.60=1.39 mmHg·min/L.
Normal values for pulmonary vascular resistance range from 0.25–1.6 mmHg·min/L.
Normal values for pulmonary vascular resistance range from 0.25–1.6 mmHg·min/L.
'''Educational objective:''' 
'''References:''' 
|AnswerA=0.72 mmHg·min/L
|AnswerA=0.72 mmHg·min/L
|AnswerAExp=Incorrect:  See calculation in the explanation
|AnswerAExp=See calculation in the explanation
|AnswerB=1.39 mmHg·min/L
|AnswerB=1.39 mmHg·min/L
|AnswerBExp=Correct:  See calculation in the explanation
|AnswerBExp=See calculation in the explanation
|AnswerC=11.25 mmHg·min/L
|AnswerC=11.25 mmHg·min/L
|AnswerCExp=Incorrect:  See calculation in the explanation
|AnswerCExp=See calculation in the explanation
|AnswerD=13.9 mmHg·min/L
|AnswerD=13.9 mmHg·min/L
|AnswerDExp=Incorrect:  This result can be achieved by dividing the difference in systolic and diastolic aortic pressures and dividing by the cardiac output (3.6L/min)
|AnswerDExp=This result can be achieved by dividing the difference in systolic and diastolic aortic pressures and dividing by the cardiac output (3.6L/min)
|AnswerE=20.83 mmHg·min/L
|AnswerE=20.83 mmHg·min/L
|AnswerEExp=Incorrect: See calculation in the explanation
|AnswerEExp=See calculation in the explanation
|EducationalObjectives=Pulmonary vascular resistance is given by the change in pressure across the pulmonary vasculature divided by the cardiac output.
|EducationalObjectives=Pulmonary vascular resistance is given by the change in pressure across the pulmonary vasculature divided by the cardiac output.
|References=First Aid 2014 page 599
|References=First Aid 2014 page 599

Latest revision as of 02:17, 28 October 2020

 
Author PageAuthor::William J Gibson (Reviewed by Serge Korjian)
Exam Type ExamType::USMLE Step 1
Main Category MainCategory::Physiology
Sub Category SubCategory::Pulmonology
Prompt [[Prompt::An attending in the cardiac intensive care unit at a major academic medical center is teaching a resident about the pulmonary vasculature. A patient with a history of hypertrophic cardiomyopathy is recovering from an anterior myocardial infarction and the following values below are measured. Which of the following corresponds to the pulmonary vascular resistance in this patient?


Pulmonary artery pressure (mean): 15 mmHg
Pulmonary capillary wedge pressure (mean): 10 mmHg
Aortic systolic pressure: 140 mmHg
Aortic diastolic pressure: 90 mmHg
Heart rate: 90/min
Stroke volume: 40 mL
]]

Answer A AnswerA::0.72 mmHg·min/L
Answer A Explanation AnswerAExp::See calculation in the explanation
Answer B AnswerB::1.39 mmHg·min/L
Answer B Explanation AnswerBExp::See calculation in the explanation
Answer C AnswerC::11.25 mmHg·min/L
Answer C Explanation AnswerCExp::See calculation in the explanation
Answer D AnswerD::13.9 mmHg·min/L
Answer D Explanation AnswerDExp::This result can be achieved by dividing the difference in systolic and diastolic aortic pressures and dividing by the cardiac output (3.6L/min)
Answer E AnswerE::20.83 mmHg·min/L
Answer E Explanation AnswerEExp::See calculation in the explanation
Right Answer RightAnswer::B
Explanation [[Explanation::The pulmonary vascular resistance of a vessel can be calculated by the change in pressure across the segment divided by the flow (R=P/Q). Generally, the pulmonary capillary wedge pressure is a good estimate of the pressure in the left atrium. Therefore, pulmonary vascular resistance is given by the formula:

Plugging in values from the above case we get the following: (15 mmHg - 10 mmHg)/(90 beats/minute*(.040 L/min))=5/3.60=1.39 mmHg·min/L. Normal values for pulmonary vascular resistance range from 0.25–1.6 mmHg·min/L.
Educational Objective: Pulmonary vascular resistance is given by the change in pressure across the pulmonary vasculature divided by the cardiac output.
References: First Aid 2014 page 599]]

Approved Approved::Yes
Keyword WBRKeyword::Lung, WBRKeyword::Vasculature, WBRKeyword::Pulmonary hypertension, WBRKeyword::Hemodynamics, WBRKeyword::Hemodynamic
Linked Question Linked::
Order in Linked Questions LinkedOrder::